How to Correctly Parameterize a Line Segment in 3D Space?

  • Thread starter Thread starter -EquinoX-
  • Start date Start date
Click For Summary
To parameterize the line segment from (1,4,-2) to (6,7,-2), the correct vector equation is \(\vec{r}(t) = (1 + 5t)\vec{i} + (4 + 3t)\vec{j} - 2\vec{k}\) for \(0 \leq t \leq 1\). There was confusion regarding the direction and components of the parameterization, particularly with the z-component, which should be constant at -2. The absence of brackets in the original equation led to misunderstandings about vector addition. Ensuring the correct format and directionality is essential for accurate results. Proper parameterization is crucial for solving such problems in 3D space.
-EquinoX-
Messages
561
Reaction score
1

Homework Statement



Parameterize C, the line segment from the point (1,4,-2) to the point (6,7,-2).

Homework Equations


The Attempt at a Solution



I got:

\vec{r}(t) = 6+5t \vec{i} + 7+3t \vec{j} + 2 \vec{k}

for some reason webassign tells me it's wrong
 
Physics news on Phys.org
Hi -EquinoX-! :smile:
-EquinoX- said:
Parameterize C, the line segment from the point (1,4,-2) to the point (6,7,-2).

I got:

\vec{r}(t) = 6+5t \vec{i} + 7+3t \vec{j} + 2 \vec{k}

erm :redface: … it's minus 2k, isn't it ? :wink:

(and some brackets would be a good idea :rolleyes:)
 


tiny-tim said:
Hi -EquinoX-! :smile:


erm :redface: … it's minus 2k, isn't it ? :wink:

(and some brackets would be a good idea :rolleyes:)


yes it's -2k, I just mistyped it.. still gives me the wrong answers
 


well, look at it this way.

let r_o=<1,4,-2>, and let u=<5,3,0>. if P1(1,4,-2) and P2(6,7,-2) then as you can see
u=P1P2
Let, r=<x,y,z>

then in general the equation of the line that passes through P1 and whose diercition is along the vector u, is

r=r_o+tu =>

r=i+4j-2k+5ti+3tj=(1+5t)i+(4+3t)j-2k

where 0=<t=<1.
 
-EquinoX- said:
… still gives me the wrong answers

In that case, either it doesn't like your absence of brackets, or since it asked for "the line segment from …", it probably expects t to be increasing in that direction. :wink:
 


-EquinoX- said:

Homework Statement



Parameterize C, the line segment from the point (1,4,-2) to the point (6,7,-2).


Homework Equations





The Attempt at a Solution



I got:

\vec{r}(t) = 6+5t \vec{i} + 7+3t \vec{j} + 2 \vec{k}

for some reason webassign tells me it's wrong
that doesn't even make sense. You are adding numbers to vectors. Do you mean
\vec{r}(t)= (6+ 5t)\vec{i}+ (7+ 3t)\vec{j}- 2\vec{k}
 
Question: A clock's minute hand has length 4 and its hour hand has length 3. What is the distance between the tips at the moment when it is increasing most rapidly?(Putnam Exam Question) Answer: Making assumption that both the hands moves at constant angular velocities, the answer is ## \sqrt{7} .## But don't you think this assumption is somewhat doubtful and wrong?

Similar threads

Replies
5
Views
2K
Replies
12
Views
2K
  • · Replies 11 ·
Replies
11
Views
3K
  • · Replies 3 ·
Replies
3
Views
3K
  • · Replies 2 ·
Replies
2
Views
2K
  • · Replies 2 ·
Replies
2
Views
1K
  • · Replies 3 ·
Replies
3
Views
2K
  • · Replies 1 ·
Replies
1
Views
2K
  • · Replies 2 ·
Replies
2
Views
4K
  • · Replies 8 ·
Replies
8
Views
2K